关于项链问题中旋转操作的置换群的循环节个数、长度的证明

对于 n n n个元素的旋转有 n n n个置换
( 0 1 . . . n − 1 1 2 . . . 0 ) ( 0 1 . . . n − 1 2 3 . . . 1 ) . . . ( 0 1 . . . n − 1 0 1 . . . n − 1 ) \begin{pmatrix} 0&1&...&n-1\\ 1&2&...&0\\ \end{pmatrix} \begin{pmatrix} 0&1&...&n-1\\ 2&3&...&1\\ \end{pmatrix} ... \begin{pmatrix} 0&1&...&n-1\\ 0&1&...&n-1\\ \end{pmatrix} (0112......n10)(0213......n11)...(0011......n1n1)
证明第 i ( 1 ≤ i ≤ n ) i(1\le i\le n) i(1in)个置换有 g c d ( n , i ) gcd(n,i) gcd(n,i)个循环节。
证:
f i ( j ) f_i(j) fi(j)表示数字 j j j在第 i i i个置换下所对应的值,则
f i ( j ) = ( j + i )    m o d    n f i ( ( j + i )    m o d    n ) = ( j + 2 i )    m o d    n . . . f i ( ( j + ( k − 1 ) i )    m o d    n ) = ( j + k i )    m o d    n = j \begin{aligned} f_i(j)&=(j+i)\;mod\;n\\ f_i((j+i)\;mod\;n)&=(j+2i)\;mod\;n\\ ...\\ f_i((j+(k-1)i)\;mod\;n)&=(j+ki)\;mod\;n=j\\ \end{aligned} fi(j)fi((j+i)modn)...fi((j+(k1)i)modn)=(j+i)modn=(j+2i)modn=(j+ki)modn=j
上面假设进行了 k k k次对应,回到了 j j j,则 j j j所在的循环节长度为 k k k,求出 k k k
j + k i ≡ j   m o d    n k i ≡ 0   m o d    n \begin{aligned} j+ki&\equiv j\ mod\;n\\ ki&\equiv0\ mod\;n \end{aligned} j+kikij modn0 modn
n ∣ k i n|ki nki,又 i ∣ k i i|ki iki,所以 L C M ( n , i ) ∣ k i LCM(n,i)|ki LCM(n,i)ki,又因为 k k k是循环节长度,要取最小值,所以 k i = L C M ( n , i ) ki=LCM(n,i) ki=LCM(n,i) k = L C M ( n , i ) i k=\frac{LCM(n,i)}{i} k=iLCM(n,i),所以对于所有的数 j j j在置换 i i i中所在的循环节长度一样,都等于 L C M ( n , i ) i \frac{LCM(n,i)}{i} iLCM(n,i),所以循环节个数为 x x x,有
x ∗ L C M ( n , i ) i = n x = n i L C M ( n , i ) x = g c d ( n , i ) \begin{aligned} x&*\frac{LCM(n,i)}{i}=n\\ x&=\frac{ni}{LCM(n,i)}\\ x&=gcd(n,i)\\ \end{aligned} xxxiLCM(n,i)=n=LCM(n,i)ni=gcd(n,i)

  • 0
    点赞
  • 0
    收藏
    觉得还不错? 一键收藏
  • 0
    评论

“相关推荐”对你有帮助么?

  • 非常没帮助
  • 没帮助
  • 一般
  • 有帮助
  • 非常有帮助
提交
评论
添加红包

请填写红包祝福语或标题

红包个数最小为10个

红包金额最低5元

当前余额3.43前往充值 >
需支付:10.00
成就一亿技术人!
领取后你会自动成为博主和红包主的粉丝 规则
hope_wisdom
发出的红包
实付
使用余额支付
点击重新获取
扫码支付
钱包余额 0

抵扣说明:

1.余额是钱包充值的虚拟货币,按照1:1的比例进行支付金额的抵扣。
2.余额无法直接购买下载,可以购买VIP、付费专栏及课程。

余额充值